Jump to content

ChiMiwhh's Content

There have been 125 items by ChiMiwhh (Search limited from 06-06-2020)



Sort by                Order  

#726892 Tại sao nhiều bài toán BĐT thế ?

Posted by ChiMiwhh on 12-05-2021 - 18:10 in Kinh nghiệm học toán

Phần lớn nó không dạy tư duy, nó dạy cách biến đổi (vì có khuôn mẫu cả rồi)
Và các nhà toán học không hề học đều đâu, vì làm thế sẽ tảu hoả nhập ma vì quá nhiều
Riêng về hình học phẳng thì tư duy của nó hoàn toàn khác với hình học hiện đại, nó cũng không tới mức tốt cho tưởng tượng
Thích điểm cao thì cứ học nhưng đừng có vụ tư duy này nọ

Về hình học phẳng thì nó cũng tốt cho suy luận ngược mà nhỉ?




#726919 Tại sao nhiều bài toán BĐT thế ?

Posted by ChiMiwhh on 12-05-2021 - 22:55 in Kinh nghiệm học toán

Có lẽ. Nhưng mình thấy hình học phẳng việt nam quá khó, tư duy để làm được nó có lẽ vượt xa cái gọi là suy luận ngược

Có lẽ khó thật nhưng mk nghĩ mọi thứ đều có thể suy luận ra được và đó là đích đến của chúng ta. Chứ không phải như vậy thì ta phải giải quyết nó như thế nào?




#726983 Tại sao nhiều bài toán BĐT thế ?

Posted by ChiMiwhh on 13-05-2021 - 22:03 in Kinh nghiệm học toán

Suy luận kiểu gì thì mình hoàn toàn chịu. Có lẽ một phần là nhớ bài. Nhưng mà tư duy có được dường như khá vô dụng hay ít nhất là không cần thiết. Lý do là không có vẫn học được toán

Mình thấy nó khá hữu dụng trong lập trình đấy chứ

Trên Ted cũng có 1 bài nói về tư duy suy ngược, của 1 kiện tướng cờ vua thì phải




#726333 [TOPIC] ÔN THI SỐ HỌC VÀO THPT CHUYÊN NĂM 2020-2021

Posted by ChiMiwhh on 01-05-2021 - 23:04 in Số học

Bài 117Tìm $x\in Z^+$ và p là số nguyên tố sao cho $7^p-4^p=31x^2$

Ngồi hay đứng thử $p=2,3$ ta có $p=3$ thỏa mãn

Xét $p>3$

TH1: $p=3k+1$ nên 

$7.343^k-4.64^k\equiv 2^k(7-4)\equiv 0(mod31)$ vô lí

tương tự TH còn lại




#724723 [TOPIC] ÔN THI SỐ HỌC VÀO THPT CHUYÊN NĂM 2020-2021

Posted by ChiMiwhh on 29-03-2021 - 16:06 in Số học

Bài 83: Cho m;n nguyên dương thỏa $m+n+1$ là 1 ước nguyên tố của $2(m^{2}+n^{2})-1$. Chứng minh mn là số chính phương.

GS $m\geq n$ 

Nên từ giả thiết 

$2(m^2+n^2)-1=(m+n)^2+(m-n)^2-1$ suy ra $(m-n)^2\vdots m+n+1$ mặt khác nó lại là số nguyên tố nên

TH1;

$m-n\geq  m+n+1$ vô lí nên suy ra 

TH2:

$m-n=0$ nên $m=n$ hay đpcm

P.s: dạo này luyện đề bận wa, chỉ kịp lên lấy vài bài về làm




#726615 [TOPIC] ÔN THI SỐ HỌC VÀO THPT CHUYÊN NĂM 2020-2021

Posted by ChiMiwhh on 08-05-2021 - 00:26 in Số học

Bài 130: Cho $a;b$ là các số nguyên dương thỏa $ab\mid a^{2}+b^{2}$  . Tìm tất cả giá trị có thể có của $\frac{a^{2}+b^{2}}{2ab}$

*Nếu được hãy giải cho trường hợp $a;b$ nguyên 

Cách 2:

Đặt $a=dx$

$b=dy$ với $d=(a,b)$

$(x,y)=1$

Chỉ ra được $x^2+y^2\vdots xy$ 

Dễ chi ra $x=y$ hay $a=b$

nên chỉ có 1 giá trị là 2

 

Cách 1 có thể giải đc Bài toán phụ 

Sao phog cứ làm cách này nhỉ?? :)




#726702 [TOPIC] ÔN THI SỐ HỌC VÀO THPT CHUYÊN NĂM 2020-2021

Posted by ChiMiwhh on 08-05-2021 - 22:29 in Số học

Là sao a? $q+1>q-2$ mà một trong hai số chia hết cho $p^3$ nên $q+1=p^3$ còn $q-2=p^2+1$ (Ý kiến riêng của em)

Ví dụ như $p^2+1=xy$ thì cũng có thể $q+1=p^3.x$ và $q-2=y$ có sai gì đâu đúng k?




#725916 [TOPIC] ÔN THI SỐ HỌC VÀO THPT CHUYÊN NĂM 2020-2021

Posted by ChiMiwhh on 25-04-2021 - 11:20 in Số học

Không mất tính TQ, giả sử $x\leq y\leq z$ 
$\Rightarrow 4^{x}(1+4^{y-x}+4^{z-x})=a^2$
th1: ngoặc bên VT lẻ thì $1+4^{y-x}+4^{z-x}=(2k+1)^2\Rightarrow 4^{y-x-1}+4^{z-x-1}=k(k+1) \Leftrightarrow 4^{y-x-1}(1+4^{z-y})$
với k chẵn thì $\left\{\begin{matrix} 4^{y-x-1}=k & \\ 4^{z-y}+1 =k+1& \end{matrix}\right.$
nên z=2y-z-1 từ đó thay vào ptgt dc scp
tương tự với trường hợp k lẻ
th2: ngoặc bên VT chẵn thì x=y hoặc z=y (vô lí)
Vậy z=2y-x-1 (TM)

Why k chẵn thì nó lại bằng như thế :))




#726700 [TOPIC] ÔN THI SỐ HỌC VÀO THPT CHUYÊN NĂM 2020-2021

Posted by ChiMiwhh on 08-05-2021 - 22:21 in Số học

Ta có: $p^3(p^2+1)=(q-2)(q+1)$

$\blacksquare $ Xét $p=2$ thì $(q-2)(q+1)=40\Rightarrow q=7$

$\blacksquare $ Xét $p=3$ thì $(q-2)(q+1)=270\Rightarrow q=17$

$\blacksquare $ Xét $p>3$ thì $(q+1)-(q-2)=3<p$ do đó $p+1$ và $p-2$ không thể cùng chia hết cho $p$

Nên chỉ có $p+1$ hoặc $p-2$ chia hết cho $p^3$ mà $p^3>p^2+1$ nên $\left\{\begin{matrix}p^3=q+1 & \\ p^2+1=q-2 & \end{matrix}\right.\Rightarrow p=2;q=7$

Vậy có 2 cặp số (p;q) thỏa mãn là (2;7) và (3;17)

Tại sao TH3 lại có thể chia ra như vậy. Như thế là không chặt chẽ bởi vì $p^2+1$ có thể là hợp số




#726645 [TOPIC] ÔN THI SỐ HỌC VÀO THPT CHUYÊN NĂM 2020-2021

Posted by ChiMiwhh on 08-05-2021 - 10:09 in Số học

$\boxed{135}$ Tìm các số nguyên dương $(p,q)$ thỏa mãn $p$ là số nguyên tố và $p^5+p^3+2=q^2-q$

P.s: Một bài hơi giống 134




#726046 [TOPIC] ÔN THI SỐ HỌC VÀO THPT CHUYÊN NĂM 2020-2021

Posted by ChiMiwhh on 28-04-2021 - 00:56 in Số học

Bài 114Tìm tất cả các số x, y $\in N$ thỏa mãn $85^x-y^4=4$

 

 

Ta có

**: x chẵn . dễ chỉ ra vô nghiệm

**: x lẻ

$85^x=(y^2+2y+2)(y^2-2y+2)$

Đặt $d=UCLN(y^2+2y+2;y^2-2y+2)$

nên $4y\vdots d$

Trường hợp d chẵn thì $85^x$ chẵn(Vô lí)

Trường hợp $y\vdots d$ thì $85^x\vdots d$ nên $4\vdots d$

Nên $d=1$

Lại có $y^2+2y+2>y^2-2y+2$

Áp dụng bổ đề 

$a^b=xy$ với $(x,y)=1$ thì $x=(x_1)^b$ và $y=(y_1)^b$ với $(x_1,y_1)=1$. Chứng minh ở topic cũ :) 

Ta có $y^2+2y+2=17^x$

$y^2-2y+2=5^x$

 

Trừ vế theo vế, ta có

$4y=17^x-5^x$

Thế lại vào phương trình gốc

với $(a,b)=(17^x,5^x)$

Ta có $(a-b)^4-1024-256ab=0$ tương đương

$(a^2-2ab+b^2+8a+8b+32)(a^2-2ab+b^2-8a-8b+32)=0$ đến đây giải dễ rồi nhỉ :)

Ko biết cách này vào phòng thi được ko  :))

Vừa chợt kiếm ra ý tưởng khi giải TH sau, Có thể chặn được x




#724855 [TOPIC] ÔN THI SỐ HỌC VÀO THPT CHUYÊN NĂM 2020-2021

Posted by ChiMiwhh on 03-04-2021 - 10:24 in Số học

Bài 91: Tìm nghiệm nguyên là các số nguyên tố $p,q$ thỏa mãn

$p+q=(p-q)^3$




#725744 [TOPIC] ÔN THI SỐ HỌC VÀO THPT CHUYÊN NĂM 2020-2021

Posted by ChiMiwhh on 21-04-2021 - 18:02 in Số học

Bài 113: Tìm tất cả các số nguyên dương $m,n$ thỏa mãn $n^2+3n+1=5^{m}$

p.s: nhìn hơi giống 112 :)




#725743 [TOPIC] ÔN THI SỐ HỌC VÀO THPT CHUYÊN NĂM 2020-2021

Posted by ChiMiwhh on 21-04-2021 - 17:57 in Số học

Bài 112Tìm x, y thuộc Z$x^4-6x^2+1=7.2^y$

Chắc phải ghi là y thuộc N

thì giải như thế này

Xét $y=0,1,2,3$ thử được $y=2$, $x=3$ hoặc $x=-3$

Xét $y\geq 4$

Tương đương với

$(x^2-3)^2=8(7.2^{y-3}+1)$

nhận xét ngoặc bên VP lẻ nên 

$VT=8.(2k+1)$ vô lí vì VT là SCP

éc quên x thuộc Z

Vậy $x=+-3$ và $y=2$




#725792 [TOPIC] ÔN THI SỐ HỌC VÀO THPT CHUYÊN NĂM 2020-2021

Posted by ChiMiwhh on 23-04-2021 - 00:06 in Số học

Bài 113: Tìm tất cả các số nguyên dương $m,n$ thỏa mãn $n^2+3n+1=5^{m}$

p.s: nhìn hơi giống 112 :)

Hic ko có ai làm

Xét $m=1$ nên $n=1$

Xét $m\geq 2$

$n^2+3n+1=5^{m}$ 

tương đương với

$(2n+3)^2=5(4.5^{m-1}+1)$

Nên ngoặc bên VP không chia hết cho 5

suy ra $(2n+3)^2$ không chia hết cho 25(Vô lí)

Vậy $m=n=1$




#725793 [TOPIC] ÔN THI SỐ HỌC VÀO THPT CHUYÊN NĂM 2020-2021

Posted by ChiMiwhh on 23-04-2021 - 00:45 in Số học

Bài 114Tìm tất cả các số x, y $\in N$ thỏa mãn $85^x-y^4=4$

Ta có

**: x chẵn . dễ chỉ ra vô nghiệm

**: x lẻ

$85^x=(y^2+2y+2)(y^2-2y+2)$

Đặt $d=UCLN(y^2+2y+2;y^2-2y+2)$

nên $4y\vdots d$

Trường hợp d chẵn thì $85^x$ chẵn(Vô lí)

Trường hợp $y\vdots d$ thì $85^x\vdots d$ nên $4\vdots d$

Nên $d=1$

Lại có $y^2+2y+2>y^2-2y+2$

Áp dụng bổ đề 

$a^b=xy$ với $(x,y)=1$ thì $x=(x_1)^b$ và $y=(y_1)^b$ với $(x_1,y_1)=1$. Chứng minh ở topic cũ :) 

Ta có $y^2+2y+2=17^x$

$y^2-2y+2=5^x$

 

Trừ vế theo vế, ta có

$4y=17^x-5^x$

Thế lại vào phương trình gốc

với $(a,b)=(17^x,5^x)$

Ta có $(a-b)^4-1024-256ab=0$ tương đương

$(a^2-2ab+b^2+8a+8b+32)(a^2-2ab+b^2-8a-8b+32)=0$ đến đây giải dễ rồi nhỉ :)

Ko biết cách này vào phòng thi được ko  :))




#727298 [MARATHON] Chuyên đề Bất đẳng thức

Posted by ChiMiwhh on 19-05-2021 - 23:06 in Bất đẳng thức và cực trị

Bài tiếp theo.

$\boxed{22}$ (Võ Quốc Bá Cẩn): Cho $a,b,c>0$. Chứng minh rằng: 

$$\frac{a^2+b^2+c^2}{ab+bc+ca}+\frac{1}{3} \frac{a^3+b^3+c^3}{abc}+ \frac{2(a+b+c)^2}{a^2+b^2+c^2}\geq 8$$

Hình như nó không chặt lắm

Đổi biến pqr thì đưa được về

$(p^2-3q)((pq+3r)(p^2-3q)+q(pq-9r))\geq 0$

Luôn đúng




#727889 [MARATHON] Chuyên đề Bất đẳng thức

Posted by ChiMiwhh on 05-06-2021 - 22:36 in Bất đẳng thức và cực trị

Lời giải: 

Đặt : $f(x)=3\sqrt[9]{\frac{a^{9}+b^{9}+c^{9}}{3}}-\sum \left ( \sqrt[10]{\frac{a^{10}+b^{10}}{2}} \right )=f(ta,tb,tc)$    với t là số thực bất kì

Nên ta chuẩn hóa : a + b + c = 3 .

Ta có :

$\left ( \sum \left ( \sqrt[10]{\frac{a^{10}+b^{10}}{2}} \right ) \right )^{10}\leq (a^{10}+b^{10}+c^{10}).3^9$             ( bất đẳng thức holder)

Mà :

$\sum a^{10}\leq \sqrt[3]{3.(\sum a)(\sum a^{9})}=\sqrt[3]{9(\sum a^{9})}$             ( bất đẳng thức holder)

Do đó :

$\left ( \sum \left ( \sqrt[10]{\frac{a^{10}+b^{10}}{2}} \right ) \right )^{10}\leq \sqrt[3]{3^{29}(\sum a^{9})}\Leftrightarrow\left ( \sum \sqrt[10]{\frac{a^{10}+b^{10}}{2}} \right )\leq 3\sqrt[30]{\frac{\sum a^{9}}{3} }$

Ta cần chứng minh : 

$\sqrt[30]{\frac{\sum a^{9}}{3}}\leq \sqrt[9]{\frac{\sum a^{9}}{3}}\Leftrightarrow \left ( \frac{\sum a^{9}}{3} \right )^{7}\geq 1\Leftrightarrow \sum a^{9}\geq 3$ (Luôn đúng vì $\frac{\sum a^{9}}{3}\geq \left ( \frac{\sum a}{3} \right )^{9}=1\Leftrightarrow \sum a^{9}\geq 3$) 

suy ra đpcm ( dấu ''='' xảy ra khi và chỉ khi a = b = c  )

Liệu có nhầm lẫn khi sử dụng Holder lần thứ 2?




#728034 [MARATHON] Chuyên đề Bất đẳng thức

Posted by ChiMiwhh on 11-06-2021 - 17:51 in Bất đẳng thức và cực trị

Cho em hỏi câu này ạ:

$\boxed{26}$ Tìm Min của $A= \sum x^2 +\frac{\sum xy}{x^2y+y^2z+z^2x}$ biết $a+b+c=3$ và a,b,c dương

 

Chỉ được dùng Cosi và Bunhia thôi ạ

Áp dụng $3(x^2y+y^2z+z^2x)\leq (x+y+z)(x^2+y^2+z^2)$ rồi dồn về $a^2+b^2+c^2$

P.s: đọc kĩ nội qui đi bạn, trong topic này chỉ có ad đc đăng bài thôi




#727895 [MARATHON] Chuyên đề Bất đẳng thức

Posted by ChiMiwhh on 06-06-2021 - 10:14 in Bất đẳng thức và cực trị

25/Cho a,b,c là các số thực ko âm thỏa mãn a+b+c=1. Chứng minh

$a^{4}+b^{4}+c^{4}+\frac{1}{8}\geq a^{3}+b^{3}+c^{3}$

Vornicu schur




#725025 [MARATHON] Chuyên đề Bất đẳng thức

Posted by ChiMiwhh on 06-04-2021 - 17:39 in Bất đẳng thức và cực trị

Chắc có gì đó nhầm lần vì điểm rơi là $b=d=2a=2c$.

Vâng ạ




#725068 [MARATHON] Chuyên đề Bất đẳng thức

Posted by ChiMiwhh on 07-04-2021 - 23:13 in Bất đẳng thức và cực trị

Bài làm rất tốt. Bây giờ ta sẽ bắt đầu thử sức với một bài khó để thử trình độ các bạn nhé:

 

$\boxed{5}$ Cho $a,b,c$ là số dương thỏa $x^2+y^2+z^2=1$. Chứng minh rằng: 

$$ \frac{x}{1-x^2} + \frac{y}{1-y^2} +  \frac{z}{1-z^2} \geq \frac{3\sqrt{3}}{2}$$ 

 

*Chú ý: Sử dụng bất đẳng thức Jensen được 2 điểm, các phương pháp khác 1 điểm

Đặt $a=\sqrt{3}x$ và tương tự nên ta có $a^2+b^2+c^2=3$

Cần cm $\sum \frac{a}{3-a^2}\geq \frac{3}{2}$

Xét bđt phụ

$\frac{a}{3-a^2}\geq \frac{a^2}{2}\Leftrightarrow (a-1)^2(a+2)\geq 0$ luôn đúng

Thiết lập tương tự và cộng lại

Xảy ra khi $x=y=z=\frac{1}{\sqrt{3}}$




#725901 [MARATHON] Chuyên đề Bất đẳng thức

Posted by ChiMiwhh on 24-04-2021 - 23:05 in Bất đẳng thức và cực trị

Bài tiếp theo nhé: 

$\boxed{17}$ Cho $a,b,c$ là các số thực dương. Chứng minh rằng: 

$$\frac{x}{\sqrt{\sqrt[4]{y}+\sqrt[4]{z}}}+\frac{y}{\sqrt{\sqrt[4]{z}+\sqrt[4]{x}}}+\frac{z}{\sqrt{\sqrt[4]{x}+\sqrt[4]{y}}}\geq \frac{\sqrt[4]{(\sqrt{x}+\sqrt{y}+\sqrt{z})^7}}{\sqrt{2\sqrt{27}}}$$

Đổi biến $(\sqrt[4]{x},\sqrt[4]{y}\sqrt[4]{z})=(a,b,c)$

Hay cần CM

$\sum_{cyc} \frac{a^4}{\sqrt{b+c}}\geq \frac{\sqrt[4]{(a^2+b^2+c^2)^7}}{\sqrt{2\sqrt{27}}}$

Áp dụng titu lemma, ta có

$LHS\geq \frac{(a^2+b^2+c^2)^2}{\sum_{cyc} \sqrt{b+c}}$

Đặt $B=\sum_{cyc} \sqrt{b+c}$

Cần CM $B\leq \sqrt{2\sqrt{27}}.\sqrt[4]{(a^2+b^2+c^2)}$

Áp dụng C-S và $a+b+c\leq \sqrt{3(a^2+b^2+c^2}$ là xong

xảy ra khi $a=b=c$




#725023 [MARATHON] Chuyên đề Bất đẳng thức

Posted by ChiMiwhh on 06-04-2021 - 16:40 in Bất đẳng thức và cực trị

Bài toán tiếp theo: 

$\boxed{2}$ Chứng minh với 4 số thực không âm bất kì $a,b,c,d$ ta có:

$(ab)^{\frac{1}{3}}+(cd)^{\frac{1}{3}} \leq [(a+c+b)(a+c+d)]^{\frac{1}{3}}$

Bài viết trích dẫn bđt trong sol https://diendantoanh...r-và-minkowski/

Thật may mắn khi phone của e có thể chuyển qua desktop mode

P.s: sr mng vì em áp dụng bđt phụ sai

IMG_20210406_163653_edit.jpg




#725007 [MARATHON] Chuyên đề Bất đẳng thức

Posted by ChiMiwhh on 05-04-2021 - 23:15 in Bất đẳng thức và cực trị

Bảng điểm:
Hoang72: 1 điểm

Bài toán tiếp theo:
$\boxed{2}$ Chứng minh với 4 số thực không âm bất kì $a,b,c,d$ ta có:
$(ab)^{\frac{1}{3}}+(cd)^{\frac{1}{3}} \geq [(a+c+b)(a+c+d)]^{\frac{1}{3}}$


Nếu như em ko nhầm thì nó sai với $a=b=c=d=1$
P.s: em dùng dth nên ko chỉnh được, lát e chỉnh lại sau ạ